K
Khách

Hãy nhập câu hỏi của bạn vào đây, nếu là tài khoản VIP, bạn sẽ được ưu tiên trả lời.

9 tháng 8 2019

2) Theo nguyên lí Dirichlet, trong ba số \(a^2-1;b^2-1;c^2-1\) có ít nhất hai số nằm cùng phía với 1.

Giả sử đó là a2 - 1 và b2 - 1. Khi đó \(\left(a^2-1\right)\left(b^2-1\right)\ge0\Leftrightarrow a^2b^2-a^2-b^2+1\ge0\)

\(\Rightarrow a^2b^2+3a^2+3b^2+9\ge4a^2+4b^2+8\)

\(\Rightarrow\left(a^2+3\right)\left(b^2+3\right)\ge4\left(a^2+b^2+2\right)\)

\(\Rightarrow\left(a^2+3\right)\left(b^2+3\right)\left(c^2+3\right)\ge4\left(a^2+b^2+1+1\right)\left(1+1+c^2+1\right)\) (2)

Mà \(4\left[\left(a^2+b^2+1+1\right)\left(1+1+c^2+1\right)\right]\ge4\left(a+b+c+1\right)^2\) (3)(Áp dụng Bunhicopxki và cái ngoặc vuông)

Từ (2) và (3) ta có đpcm.

Sai thì chịu

9 tháng 8 2019

Xí quên bài 2 b:v

b) Không mất tính tổng quát, giả sử \(\left(a^2-\frac{1}{4}\right)\left(b^2-\frac{1}{4}\right)\ge0\)

Suy ra \(a^2b^2-\frac{1}{4}a^2-\frac{1}{4}b^2+\frac{1}{16}\ge0\)

\(\Rightarrow a^2b^2+a^2+b^2+1\ge\frac{5}{4}a^2+\frac{5}{4}b^2+\frac{15}{16}\)

Hay \(\left(a^2+1\right)\left(b^2+1\right)\ge\frac{5}{4}\left(a^2+b^2+\frac{3}{4}\right)\)

Suy ra \(\left(a^2+1\right)\left(b^2+1\right)\left(c^2+1\right)\ge\frac{5}{4}\left(a^2+b^2+\frac{1}{4}+\frac{1}{2}\right)\left(\frac{1}{4}+\frac{1}{4}+c^2+\frac{1}{2}\right)\)

\(\ge\frac{5}{4}\left(\frac{1}{2}a+\frac{1}{2}b+\frac{1}{2}c+\frac{1}{2}\right)^2=\frac{5}{16}\left(a+b+c+1\right)^2\) (Bunhiacopxki) (đpcm)

Đẳng thức xảy ra khi \(a=b=c=\frac{1}{2}\)

22 tháng 3 2019

Áp dụng bất đẳng thức Cô-si ta có:

\(\dfrac{a^2}{b^3}+\dfrac{1}{a}+\dfrac{1}{a}\ge\sqrt[3]{\dfrac{a^2}{b^3}.\dfrac{1}{a}.\dfrac{1}{a}}=\dfrac{3}{b}\)

\(\dfrac{c^2}{a^3}+\dfrac{1}{c}+\dfrac{1}{c}\ge\sqrt[3]{\dfrac{c^2}{a^3}.\dfrac{1}{c}.\dfrac{1}{c}}=\dfrac{3}{a}\)

\(\dfrac{c^2}{a^3}+\dfrac{1}{c}+\dfrac{1}{c}\ge\sqrt[3]{\dfrac{c^2}{a^3}.\dfrac{1}{c}.\dfrac{1}{c}}=\dfrac{3}{a}\)

Cộng theo vế ta được:

\(\dfrac{a^2}{b^3}+\dfrac{b^2}{c^3}+\dfrac{a^2}{a^3}+\dfrac{2}{a}+\dfrac{2}{b}+\dfrac{2}{c}\ge3\left(\dfrac{1}{a}+\dfrac{1}{b}+\dfrac{1}{c}\right)\)

\(\Leftrightarrow\dfrac{a^2}{b^3}+\dfrac{b^2}{c^3}+\dfrac{c^2}{a^3}\ge\dfrac{1}{a}+\dfrac{1}{b}+\dfrac{1}{c}\)

2 tháng 7 2019

Áp dụng Svac

\(\Sigma\frac{a^3}{b+c}=\Sigma\frac{a^4}{ab+ac}\ge\frac{\left(a^2+b^2+c^2\right)^2}{2\left(ab+bc+ca\right)}\ge\frac{\left(a^2+b^2+c^2\right)^2}{2\left(a^2+b^2+c^2\right)}=\frac{1}{2}\left(a^2+b^2+c^2\right)\)

"=" tại a=b=c

4 tháng 7 2019

E thử làm cách khác ạ:))

Không mất tính tổng quát,giả sử \(a\ge b\ge c\)

\(\Rightarrow\hept{\begin{cases}a^2\ge b^2\ge c^2\\\frac{a}{b+c}\ge\frac{b}{a+c}\ge\frac{c}{a+b}\end{cases}}\)

Áp dụng BĐT Trebysev ta có:

\(a^2\cdot\frac{a}{b+c}+b^2\cdot\frac{b}{a+c}+c^2\cdot\frac{c}{a+b}\ge\frac{a^2+b^2+c^2}{3}\cdot\left(\frac{a}{b+c}+\frac{b}{c+a}+\frac{c}{a+b}\right)\)

\(\ge\frac{a^2+b^2+c^2}{3}\cdot\frac{3}{2}\left(nesbitt\right)\)

\(=\frac{a^2+b^2+c^2}{2}\)

Dấu "=" xảy ra khi  \(a=b=c\)


 

15 tháng 12 2016

mình nghĩ đề bài sai một chỗ :\(\frac{a^2}{b^2}\)chứ ko phải là \(\frac{a}{b^2}\)

10 tháng 5 2017

khó quá chưa học

18 tháng 8 2019

By Cauchy-Schwarz, we have:

\(VT\ge\frac{\left(a^3+b^3+c^3\right)^2}{2\left(a^3+b^3+c^3\right)+a^2b+b^2c+c^2a}\)

We will prove: \(a^2b+b^2c+c^2a\le a^3+b^3+c^3\)

\(\Leftrightarrow a^2b+b^2c+c^2a+3abc\le a^3+b^3+c^3+3abc\)

By Schur, we have: \(RHS\ge ab\left(a+b\right)+bc\left(b+c\right)+ca\left(a\right)\)

So we're only need to prove: \(ab\left(a+b\right)+bc\left(b+c\right)+ca\left(c+a\right)\ge a^2b+b^2c+c^2a+3abc\)

\(\Leftrightarrow ab^2+bc^2+ca^2\ge3abc\)

It is true by AM-GM ineq', so we have Q.E.D.

P/s: Em thử giải bài này bằng tiếng Anh (để tự luyện kĩ năng tiếng anh, tí em giải lại theo tiếng việt)

18 tháng 8 2019

Ấy nhầm:V

By Schur, we have \(RHS\ge ab\left(a+b\right)+bc\left(b+c\right)+ca\left(c+a\right)\)

So we're only need to prove \(ab\left(a+b\right)+bc\left(b+c\right)+ca\left(c+a\right)\ge a^2b+b^2c+c^2a\)

Còn lại y chang:v

AH
Akai Haruma
Giáo viên
30 tháng 7

Lời giải:

Áp dụng BĐT AM-GM:
$\text{VT}=\sum \frac{a^3}{a^2+b^2}=\sum (a-\frac{ab^2}{a^2+b^2})$

$=\sum a-\sum \frac{ab^2}{a^2+b^2}$

$\geq \sum a-\sum \frac{ab^2}{2ab}=\sum a-\sum \frac{b}{2}=\frac{a+b+c}{2}$

Ta có đpcm

Dấu "=" xảy ra khi $a=b=c$

8 tháng 5 2018

\(\frac{a^2}{b^3}+\frac{b^2}{c^3}+\frac{c^2}{a^3}=\frac{1}{b}+\frac{1}{c}+\frac{1}{a}\)

=> \(\frac{a^2}{b^3}+\frac{b^2}{c^3}+\frac{c^2}{a^3}=\frac{1}{a}+\frac{1}{b}+\frac{1}{c}\)

20 tháng 5 2018

\(\frac{a^2}{b^3}+\frac{1}{a}+\frac{1}{a}\ge3\cdot\frac{1}{b}\)

28 tháng 11 2019

Bài này đăng nhiều trên OLM rồi, lời giải vắn tắt:

\(VT=\Sigma_{cyc}\frac{a}{1+b^2}=\Sigma_{cyc}\left(a-\frac{ab^2}{1+b^2}\right)=3-\Sigma_{cyc}\frac{ab^2}{1+b^2}\)

\(\ge3-\Sigma_{cyc}\frac{ab}{2}\ge3-\frac{\frac{\left(a+b+c\right)^2}{3}}{2}=\frac{3}{2}\)

Đẳng thức xảy ra khi a = b = c = 1

28 tháng 11 2019

Ta có: \(\frac{a}{1+b^2}=a-\frac{ab^2}{1+b^2}\ge a-\frac{ab^2}{2b}=a-\frac{ab}{2}\)(bđt cô - si)

Tương tự ta có: \(\frac{b}{1+c^2}\ge b-\frac{bc}{2}\);\(\frac{c}{1+a^2}\ge c-\frac{ca}{2}\)

Cộng từng vế của các bđt trên:

\(\frac{a}{1+b^2}\)\(+\frac{b}{1+c^2}\)\(+\frac{c}{1+a^2}\)\(\ge a+b+c-\frac{ab+bc+ca}{2}\)

Dễ c/m:  \(\left(a+b+c\right)^2\ge3\left(ab+bc+ca\right)\)

\(\Rightarrow3^2\ge3\left(ab+bc+ca\right)\)

\(\Rightarrow ab+bc+ca\le3\)

\(BĐT\ge3-\frac{3}{2}=\frac{3}{2}\)

hay \(\frac{a}{1+b^2}\)\(+\frac{b}{1+c^2}\)\(+\frac{c}{1+a^2}\)\(\ge\frac{3}{2}\)

(Dấu "="\(\Leftrightarrow a=b=1\))

12 tháng 2 2017

Xét: \(\frac{a^3}{a^2+b^2}+\frac{b^3}{b^2+c^2}+\frac{c^3}{c^2+d^2}+\frac{d^3}{d^2+a^2}\)

\(\Leftrightarrow a-\frac{ab^2}{a^2+b^2}+b-\frac{bc^2}{b^2+c^2}+c-\frac{cd^2}{c^2+d^2}+d-\frac{da^2}{d^2+a^2}\)

Áp dụng bất đẳng thức Cauchy cho 2 bộ số thực không âm

\(\Rightarrow\left\{\begin{matrix}a^2+b^2\ge2\sqrt{a^2b^2}=2ab\\b^2+c^2\ge2\sqrt{b^2c^2}=2bc\\c^2+d^2\ge2\sqrt{c^2d^2}=2cd\\d^2+a^2\ge2\sqrt{d^2a^2}=2da\end{matrix}\right.\)

\(\Rightarrow\left\{\begin{matrix}\frac{ab^2}{a^2+b^2}\le\frac{ab^2}{2ab}=\frac{b}{2}\\\frac{bc^2}{b^2+c^2}\le\frac{bc^2}{2bc}=\frac{c}{2}\\\frac{cd^2}{c^2+d^2}\le\frac{cd^2}{2cd}=\frac{d}{2}\\\frac{da^2}{d^2+a^2}\le\frac{da^2}{2da}=\frac{a}{2}\end{matrix}\right.\)

\(\Rightarrow\left\{\begin{matrix}a-\frac{ab^2}{a^2+b^2}\ge a-\frac{b}{2}\\b-\frac{bc^2}{b^2+c^2}\ge b-\frac{c}{2}\\c-\frac{cd^2}{c^2+d^2}\ge c-\frac{d}{2}\\d-\frac{da^2}{d^2+a^2}\ge d-\frac{a}{2}\end{matrix}\right.\)

\(\Rightarrow a-\frac{ab^2}{a^2+b^2}+b-\frac{bc^2}{b^2+c^2}+c-\frac{cd^2}{c^2+d^2}+d-\frac{da^2}{d^2+a^2}\ge a+b+c+d-\frac{a}{2}-\frac{b}{2}-\frac{c}{2}-\frac{d}{2}\)

\(\Rightarrow a-\frac{ab^2}{a^2+b^2}+b-\frac{bc^2}{b^2+c^2}+c-\frac{cd^2}{c^2+d^2}+d-\frac{da^2}{d^2+a^2}\ge\frac{a+b+c+d}{2}\)

\(\Leftrightarrow\frac{a^3}{a^2+b^2}+\frac{b^3}{b^2+c^2}+\frac{c^3}{c^2+d^2}+\frac{d^3}{d^2+a^2}\ge\frac{a+b+c+d}{2}\) ( đpcm )

12 tháng 2 2017

Cách của bạn Minh dài quá mình xin làm cách ngắn hơn:

Đầu tiên ta chứng minh bổ đề:

\(\frac{x^3}{x^2+y^2}\ge\frac{2x-y}{2}\)

\(\Leftrightarrow2x^3-\left(x^2+y^2\right)\left(2x-y\right)\ge0\)

\(\Leftrightarrow y\left(y-x\right)^2\ge0\)(đúng)

Từ đó ta có: \(\left\{\begin{matrix}\frac{a^3}{a^2+b^2}\ge\frac{2a-b}{2}\\\frac{b^3}{b^2+c^2}\ge\frac{2b-c}{2}\\\frac{c^3}{c^2+d^2}\ge\frac{2c-d}{2}\\\frac{d^3}{d^2+a^2}\ge\frac{2d-a}{2}\end{matrix}\right.\)

Cộng 4 cái trên vế theo vế ta được

\(\frac{a^3}{a^2+b^2}+\frac{b^3}{b^2+c^2}+\frac{c^3}{c^2+d^2}+\frac{d^3}{d^2+a^2}\ge\frac{2a-b}{2}+\frac{2b-c}{2}+\frac{2c-d}{2}+\frac{2d-a}{2}=\frac{a+b+c+d}{2}\)